Diskussion:Graph einer Funktion/Riemannsche Untermannigfaltigkeit des R^n/Volumenform/Fakt

Seiteninhalte werden in anderen Sprachen nicht unterstützt.
Aus Wikiversity

Ich glaube das ² der Summe ist nicht ganz eindeutig. Müsste es nicht näher an den Summanden stehen?